Cực trị trên tập rời rạc

Một phần của tài liệu Tài liệu bồi dưỡng học sinh giỏi Toán phần 2 (Trang 69 - 74)

. Đối với một bài toán có điều kiện, các trường hợp đặc biệt xảy ra khi các biến có mặt bằng nhau hoặc xảy ra dấu bằng trong các đánh giá của điều kiện

7Cực trị trên tập rời rạc

7.1 Phương pháp xây dựng cấu trúc của điểm mà tại đó đạt Min, Max Min, Max

Dự đoán Min, Max của biểu thức Ặ(z) đạt tại z" nào đó uà chứng minh dụ đoán

đó.

Trong phương pháp nàu thường ta phải sử dụng các kiến thúc của Số học, Tổ hợp và Toán rời rạc. Việc dự đoán thường được thực hiện bởi phép quụ nạp dựa uào một số tắnh toán ban đầu.

7.2 Vắ dụ minh hoạ

Vắ dụ 1. Cho một n - giác lồi (n > 4), trơng đó khỡng có ba đường chéo nào đồng quụ tại một điểm nằm trong đa giác. Gọi Ặ(n) là số các đường chéo có thể kẻ được sao cho các phần mà chúng cắt ra khỏi đa giác đã cho đều là các tam giác.

Hãy xác định Max f(n).

Lời giải. Gợi Ặ(n) là số các đường chéo có thể kẻ được thoả mãn yêu cầu bài ra. Xét

một số trường hợp ban đầu của n. Bằng cách uẽ hành cụ thể ta được n= J4|2|6|17|168|19|112 Ặ(n)< | 2|2|1 5161681911 Từ đó ta có dự đoán fắn)<|Ỳ | Ở4 69

Trơng đó ta ký hiệu [x] là số nguyên lớn nhất không uượt quá z. Để chứng. mứnh (*) trước hết ta chứng minh

Bồ đề: Nếu tập Ế các đường chéo của một n - giác lồi (n > 5) chia n - giác đó thành các tam giác thà có ắt nhất một đường chéo Ạ S không cắt tắt cả các đường chéo khác Ạ S tại điểm trong của n - giác đã cho.

Chứng minh: Xét các hình uẽ sau:

Hình 1. Hình 2. Hành 4.

Từ các hành uẽ trên ta có các nhộn xét biển nhiên sau:

1) Không có đường chéo nào Ạ S cắt > 2 đường chéo Ạ S9 khác (xem Hình 1). Do hai góc Ủ; Ô8cóỦ+jỪ3 180Ợ ).

2) Giả sử mọi đường chéo e S đều cắt đúng một đường chéo Ạ S. Xét hai đường chéo Ạ Ế là AC ; BD cắt nhau tại O (xem Hình 9) ). Do n > 5 nên có ắt nhất một cạnh của tứ giác ABCƠD là đường chéo của n - giác đã cho. Giả sử đó là cẤnh AB. Tn có ABẠ 5 tì nếu không thà phần n - giác với biên chứa đường gấp khúc AOB sẽ không là tan giác nên sẽ có một đường chéo e S cắt AB, khi dó đường chéo đó cồn cắt hoặc AC, hoặc BD. Điều đó trái uới nhận xét 1) (Có một đường chéo Ạ 6 cắt hai đường chéo Ạ S).

Từ hai nhận sét trên suy ra kết luận của bổ đề

Trở lại bài toán đang xét. Hiển nhiên, (*Ộ) đã đúng uới n = 4;5;6. Giả sử (*) đúng uới mọi R, đ S k < n, n > 6. Theo bổ đề, tồn tại một đường chéo Ạ Ế không cắt các đường chéo khác Ạ 9 uà chia n - giác đã cho thành hai phần rời nhau. Giả sử đường

chéo đó là Á\A, (xem Hành ở) uà nó chia n - giác đã cho thành ?P - giác 0à q - giác (uới J&?g<n; p+q=n+2).

khi đó, số Ƒ(n) các đường chéo kẻ được thoả mãn yêu cầu bài ra là

f) =1+ Ặ() + Ặ().

Theo giả thiết quy nạp ta có

*Ị lÌ ỞỞn rỞỞỞỞỞ ỞỞỞ e5 Bà) "ỞỞI | Hn M[# SỊ t_ỞỞI Ị d> Vậu (*) cừng đúng tới n.

Đo đó, theo nguyên lú quụ nạp, (*) đứng uới mọi n GẠ ứ; n > 4. Vậu số lớn nhất các ; ể

` . ở

đường chéo kẻ được thoả mãn êu câu bài ra là 5) Ở4.

Vắ dụ 2. (VMO 2003) Tìm số n Ạ ứ lớn nhất sao cho hệ phương trình sau có

nghiệm nguyên.

Ể + DĨ + ti = (+2)? + tệ = (ụ +3)? + gỆ = --- = (+)? + về, (1) (adsbygoogle = window.adsbygoogle || []).push({});

Lời giải. Hiển nhiên nếu hệ (1) không có nghiệm (# ;¡ 01 ¡ 2 ;-** ;¡ Ua) nguyên khi n = k thì nó cũng không có nghiệm nguyên uới n > k.

Với n = 3 dễ thấu hệ có nghiệm: (Z ; tì; 9a; 9ã) =(Ở2; 0; 1; 0). 1a sẽ chứng minh uới n = 4 thì hệ (1) không có các nghiệm nguyên. Thật uậu, giả sử tồn tại các số # ;1ì ; 12; 9a; ta Z uà thoả mãn:

(z+ 1L +ưi =(+2)?+2 =(z+3)?+=(e+4)2+2. (2)

Từ (2), do z+ 1 uà z+3 cùng tắnh chẵn, lẻ nên Uị uà a cùng tắnh chẵn, lẻ. TT: ương tự, Ua Uầ tạ cùng tắnh chăn, lẻ.

Doz+1 tuà++2 khác tắnh chẵn, lẻ nên Uị 0à ; khác tắnh chẵn, lẻ. Tương tự, U3 Uồ ta khác tắnh chẵn, lẻ.

Nhận xét rằng:

Với aẠ Z2; a lẻ thà aỢ= 1 (mod 8), cồn uới a Ạ Z.; a chẵn thì 2a? = (mod 8). Từ (2) ta có:

(z + 1)Ợ + vĩ + (+ + 3)? + yệ = 2((z + 2)? + ệ ẹ 2V) = 9?+vỆ +2. (*) (+ +2)Ợ + y2 + (Ủ + 4)? + y2 = 2Í(s + 3)2 + Ệ ẹ 2W) = v2 + 2+2. (x*) (+ +2)Ợ + y2 + (Ủ + 4)? + y2 = 2Í(s + 3)2 + Ệ ẹ 2W) = v2 + 2+2. (x*)

Nếu ụạ chăn thì theo trên ta có tu;ts lẻ, uà từ (*) ta được: 0=(1+1+2) (mod 8) : ô lý. Nếu ụỈ lẻ thà theo trên ta có ụạ lẻ ; a chẵn tà từ (**) ta được:

0=(1+1+2) (modệ8) : 0ô lý.

Vậu giả sử ban đầu là sưi, tức là uới n = 4 thà hệ (1) không có các nghiệm nguyên. Đo đó m = 3 là giá trị lớn nhất để hệ phương trành đã cho có nghiệm Ạ Z.

Vắ dụ 3. Chon 3 3 tà các số 34;Z2;--- ;#nẤ_ Ạ ứ, biến thiên uà thoả mãn,

.(ụĐ

1) #2 +3 +: Đan =n

2) gì + 2a + 3# +--- + (n Ở 1)#ẤỞ-¡ = 2n Ở 2

mrỞ]

Tầm Min Ê uới Ặ = Ặ(đ1;#a;- + ;#n_t) :Ở À >k.x,.(2n Ở k). k=1

Lời giải. Wì tập giá trị của Ặ là tập con của ứ nên tồn tại Minf. Giả sử Minf = f(xỢ) = Í(xỉ;Xã;--' ¡XỶ ).

- Tù điều kiện 2) suy ra z;_¡ <2. Dấu bằng xắu ra khi uà chỉ khi #] = ử5 =''' =z]n_Ỉ=0=n = 2 (theo 1) ). #] = ử5 =''' =z]n_Ỉ=0=n = 2 (theo 1) ).

Đó là điều uô lý uì n > 3. Vậy zẬ_; <S 1 = z?_, Ạ {0;1}. +) Nếu z}_ = 0 thà 1) #1 + #2 +23 + co 88a =T () 2) zị + 23 + 3z + -:: +(nỞ2)z⁄_ạ=2nỞ2 ` a) Nếu 3m Ạ 1..nỞ 2, +? >0 còn zẨ =0, Vizmn th Đa Ôn T rnzmẤ = 2n Ở 2 => ?nn = 2n Ở 2 = n(2 Ởm) =2 : vô lý don > 3.

b) Nếu 3i, j, 1<i<j<nỞ2,7>Ậi+2. sao cho zẬ, z7 > 0. Xét bộ số Ủ' := (1;g;-'- ¡2 _¡) tác định như sau

*

1= 1E. khik # {j;1+1;7+1}

¡=8f 1; địa =địa |; =7 1; đi=27 +]

Dễ dàng kiểm tra rằng bộ các số #' := (1; +); --- ah _\) ác định như uậu thoả mãn điều

kiện (i). Nhưng \

(1; #3; +! 88-1) Ở Ặ(1i đại cỲn ;8m_j) = sỉ: = 2 Ở 7) >0

nên Ặ không đạt Min tại +" = (z1;13;--- ¡z2 1). (adsbygoogle = window.adsbygoogle || []).push({});

Vậu ta phổi có: 3i, x7, rị.\ > 0 0à zx =0, Vkz#¡, 2+1. Khi đó

(0 Ủ J7: Tắn =n Ư#ỉ + (+.1)Z7,¡ = 2n Ở 2 = z1 =(9Ở in Ở9. Do z?.¡>03>Ư<2=¡= 1. Vậy z3 =n Ở 2, #T = 2 tà Ặ(") = Ặ(2¡n Ở 2;0;0;--- ;0) = 2(2n Ở 1) + 2(n Ở 2)(2n Ở 2) = An? Ở 8n + 6. +) Nếu +z}_¡ = 0 thì ()ẹ 1) zị + zỌ +73 +. +2 2=nỞl 2) 7 +23 + 35 + --- +(nỞ 2)Z}_ỞƯ =mỞ 1 => 1 = đạ =''' =7) =0, 21 =nTỞ Ì = Ặ(zỢ) = 1.(m Ở 1)(9n Ở 1) + (n ~ 1).1.(2n Ở n + 1) = 3n Ở ần. Nhưng tới mọi n > 3 ta luôn có

(4n Ở 8n + 6) Ở (3n? Ở 3n) = (n Ở 3)(n Ở 2) > 0

+

nên ta được kết quả sau

Min f = 3n? Ở 3n = f(n Ở 1;0;0;--- ;0; 1).

Vắ dụ 4. Hàm số (Ặ(n)) : ứ* Ởể R szác định như sau n{n + 1)

/0)=0; Ặn)=/(E)+(1) 2 ,n>2

1) Xác định Min; Max của Ặ(n) uới n < 1999 tà tm tắt cả các giá trị n Ạ 1..1998 để Ƒ(n) đạt Min; Max.

8) Có bao nhiêu giá trị n Ạ 1..1999 để Ặ(n) = Lời giải. To có

SH

I=

f0 =/(ỌJ)=(U) 2 =

Nhận xét rằng trong hệ cơ số 2, nếu n = ự1ỏa'::ũy-aảy-iảy thì n

B =Ở= 60162 ''' đkỞ26kỞ1-

Ngoài ra

n=0;3 (mod 4) ẹ ãy-1ãy = ŨŨ hoặc 11. n=1;2 (mod 4) ẹ ụg-i8y =ÚI hoặc lỗ.

Như uậu:

Ặ(n) tăng 1 đơn tị đối tới /S1) nếu T. = ự1ự2':.0pg-a2ựy- lũy UỚI 0y = đỞ1. Ƒ(n) giảm 1 đơn tị đối uới /5)) niỄu = ựj8a--'ựy-a2ựy- 10x UỚI dy # ữyỞ1.

Ký hiệu u(n) là số cặp 00 ; 11 cờn 0(n) là số cặp 01 ; 10 trong cách uiết theo cơ số 9 của số n. Tu sẽ chứng mình

Ƒ(n) = u(n) Ở 0(n), VneẠeứ". (+)

Tu sẽ chứng mưnh ( ") bằng phương pháp quụ nạp toán học theo k là số chữ số trong cách uiết nhị phân của số n.

Thật uậu, khi k = 1 = n = 1 th Ặ(1) =0; u(1) = 0u(L) = 0 => (x) đúng tới k = 1. Nếu k = 2 thì

+) Hoặc n = 2 = 10a => u(2) =0; 0(2) = 1 còn Ƒ(2) = Ở1 => Ặ(2) = u(2) Ở 9Q). +) Hoặc n = 3 = 11a + u(3) =1; 0(3) = 0 cờn

Ặ@) = Ặ) +1=1= /(3) = u(3) Ở 0(3).

Giả sử (*) đứng tới k, hay là (*) đúng uới mọi số n = B182 dg-a0y-lũk.

Xét mm = G162 -'' GyỞa2đrỞ10kÔk+1- m + ` Nếu a,+¡ = 0 = [z]=ệ mn + 1 lẻ tà Q06 cs20y các Sa ÁGdUÃSXE (6u 262g/&GOvobuic taxi uyên

Nếu aẤ = 0 thì n chăn uà (adsbygoogle = window.adsbygoogle || []).push({});

u(m) = u(n) +1; 0(m) = u(n); (Ở1)"0#Đ =1 = Ặứm) = Ặ(n) + 1 = u(n) Ở u(n) +1 = u{(m) Ở 0(mì. = Ặứm) = Ặ(n) + 1 = u(n) Ở u(n) +1 = u{(m) Ở 0(mì. Nếu ay = 1 thà n lẻ 0ò

u(m) = u(n) ¡ 0(m) = o(n) +1; (~1)"09) = Ở1

= Ặ(m) = Ặ(n) Ở 1 = u(w) Ở u(n) Ở 11 = u(m) Ở u(m).

Nếu a;+¡ = 0 thà lập luận tương tự ta cùng được Ặ(m) = u(m) Ở 0(m). Vậu (*) cũng đứng tới k + 1 nên theo nguyên lú quy nạp ta được (*) đứng uới mọi n Ạ ứ".

Trở lại bài toán đã cho

1) Từ (*) ta thấu để Ặ(n) lớn nhất thì u(n) phải nhỏ nhất. Mà 0ắn) > 0 nên ta phải tìm số n. lớn nhất, < 1999 sao cho 0(n) = 0. Nhận xét rằng

11111111111; = 2047 > 1999; 1111111111 = 1023 < 1999

nên. số rỈ lớn nhất Ạ 1..1985 có uắn) nhỏ nhất là số 1029. Khi đó Ặ(n) = 9Ở 0 =9. Vậy ¡ Ma Ặf(n) =9 = Ặ(1023).

Để Ặ(n) nhỏ nhất ta phải có u(n) nhỏ nhất, đồng thời u(n) lớn nhất. Lập luận tương tự trên ia được số 10101010101; = 1365 là số có u(n) = 0 là nhỏ nhất và 0(m) = 10 là lớn

Một phần của tài liệu Tài liệu bồi dưỡng học sinh giỏi Toán phần 2 (Trang 69 - 74)